Computar: $$\prod_{n=1}^{\infty}\left(1+\frac{1}{2^n}\right)$$
Yo y mi amigo nos encontramos con este producto. ¿Es el producto hasta el infinito igual a $1$ ?
Si no, ¿cuál es la respuesta?
Computar: $$\prod_{n=1}^{\infty}\left(1+\frac{1}{2^n}\right)$$
Yo y mi amigo nos encontramos con este producto. ¿Es el producto hasta el infinito igual a $1$ ?
Si no, ¿cuál es la respuesta?
¿Es el producto hasta el infinito igual a $1$ ?
No hay duda de que no. Todos los términos individuales son mayores que $1$ . Así que si los multiplicas juntos, siempre estarás aumentando y no podrás volver a $1$ .
Si no, ¿cuál es la respuesta?
El producto en cuestión es $$ \prod_{n=1}^\infty (1 + x^n) $$ donde $x = \frac12$ . Este producto equivale a $$ \sum_{n=0}^\infty q(n) x^n $$ donde $q(n)$ es el número de particiones de $n$ en partes distintas (cada parte $\ge 1$ ), y también es igual a $$ \prod_{n=1}^\infty \frac{1}{1 - x^{2n-1}} = \frac{\Phi(x^2)}{\Phi(x)} $$ (ver Wikipedia ), donde aquí $\Phi$ es la función de Euler no debe confundirse con la función totiente de Euler. Así que su producto es igual a $$ \boxed{\frac{\Phi(1/4)}{\Phi(1/2)} = 2.38423\ldots}. $$ No espero que esto se pueda simplificar.
¿Puede ver su pregunta actualizada e intentar responder de la misma manera? Estoy muy interesado.
@WinVineeth Supongo que te refieres al producto diferente que surgió en los comentarios, que fue el problema del que el OP sacó esto. Es decir $2$ y se responde aquí .
En el enlace que has proporcionado, el producto parece ser $$\prod_{n=0}^{\infty} \left (1+\frac{1}{2^{2^n}}\right)$$
Tenga en cuenta que $$(1-\frac{1}{2})\prod_{n=0}^{\infty} (1+\frac{1}{2^{2^n}})=\lim_{n \to \infty} 1-\frac{1}{2^{2^{n+1}}}=1$$ Por el hecho de que $$(1+\frac{1}{2^{2^k}})(1-\frac{1}{2^{2^k}})=1-\frac{1}{2^{2^{k+1}}}$$
$$\prod_{n\geq 1}\left(1+\frac{1}{2^n}\right)=\exp\sum_{n\geq 1}\sum_{m\geq 1}\frac{(-1)^{m+1}}{m 2^{mn}}=\exp\sum_{m\geq 1}\frac{(-1)^{m+1}}{m(2^m-1)} $$ donde la última serie es (rápidamente) convergente por la prueba de Leibniz. Se deduce que el producto original está entre $\exp\left(\frac{121}{140}\right)$ y $\exp\left(\frac{3779}{4340}\right)$ . Truncando la serie mostrada en $m=30$ obtenemos $$\prod_{n\geq 1}\left(1+\frac{1}{2^n}\right)\approx 2.384231029.$$
Otra forma de obtener la misma respuesta de 6005. Si tomamos el logaritmo de este producto y la expansión de Taylor del logaritmo obtenemos $$\sum_{n\geq1}\sum_{k\geq1}\frac{\left(-1\right)^{k+1}}{k2^{mk}}=\sum_{k\geq1}\frac{\left(-1\right)^{k}}{k\left(1-2^{k}\right)}=-\sum_{k\geq1}\frac{\left(-1\right)^{k}}{k2^{k}\left(1-\frac{1}{2^{k}}\right)} $$ $$=-\sum_{k\geq1}\frac{1}{k4^{k}\left(1-\frac{1}{4^{k}}\right)}+\sum_{k\geq1}\frac{1}{k2^{k}\left(1-\frac{1}{2^{k}}\right)} $$ y como tenemos $$\log\left(\Phi\left(q\right)\right)=-\sum_{k\geq1}\frac{q^{n}}{k\left(1-q^{n}\right)} $$ donde $\Phi\left(q\right) $ es el Función de Euler tenemos $$\sum_{n\geq1}\sum_{k\geq1}\frac{\left(-1\right)^{k+1}}{k2^{mk}}=\log\left(\frac{\Phi\left(1/4\right)}{\Phi\left(1/2\right)}\right). $$ Tal y como se escribió en 6005, probablemente no haya ninguna simplificación para este resultado. El resultado se puede escribir también como Símbolo q-Pochhammer $$\prod_{n\geq1}\left(1+\frac{1}{2^{n}}\right)=\left(-1;\frac{1}{2}\right)_{\infty}.$$
Esta no es una respuesta completa. Ya hay respuestas completas publicadas.
Puedo demostrar que la secuencia $x_n=\prod_{i=1}^n\left(1+\frac{1}{2^i}\right)$ converge, es estrictamente creciente y menor que $e$ Así que $\prod_{i=1}^{+\infty}\left(1+\frac{1}{2^i}\right)\le e$ . Observe que $x_n>0$ , $\forall n$ Así que $x_{n+1}=x_n\left(1+\frac{1}{2^n}\right)>x_n$ , $\forall n$ . $$x_1=1.5\le x_n=e^{\ln\prod_{i=1}^{n}\left(1+\frac{1}{2^i}\right)}=$$
$\ln\left(1+\frac{1}{n}\right)<\frac{1}{n}$ , $\forall n\ge 1$ , $n\in\mathbb Z$ es cierto. Una desigualdad más general $e^x> x+1$ , $\forall x\in\mathbb R$ , $x\neq 0$ también es cierto.
$$=e^{\sum_{i=1}^n \ln\left(1+\frac{1}{2^i}\right)}<e^{\sum_{i=1}^n \frac{1}{2^i}}<e^{\sum_{i=1}^{+\infty}\frac{1}{2^i}}=e$$
WolframAlpha dice que,+j%3D1+to+infinity) el producto infinito es $\approx 2.384$ .
I-Ciencias es una comunidad de estudiantes y amantes de la ciencia en la que puedes resolver tus problemas y dudas.
Puedes consultar las preguntas de otros usuarios, hacer tus propias preguntas o resolver las de los demás.
1 votos
Wolfram Alpha cree que el resultado es $\approx 2.384231029031371724149899$ .
0 votos
¿No era el problema algo parecido a esto? math.stackexchange.com/questions/1183416/ Porque la respuesta debería ser ninguna de las anteriores.
2 votos
Solución: math.stackexchange.com/questions/1922298/
0 votos
WA dice: $\prod_{n=0}^\infty(1+\frac{1}{2^{2^n}})=2$